0
$\begingroup$

Let $$G=\langle x, y, z\mid xyx^{-1}=zy, xzx^{-1}=z, yz=zy\rangle,$$ denote $l^1(G)^{\times}$ to be the set of units in $l^1(G)$, which we have considered as a ring with multiplication defined by the usual convolution, i.e., $(\sum_{g\in G}\lambda_gg)(\sum_{h\in G}\mu_hh)=\sum_{g, h\in G}\lambda_g\mu_hgh$.

Can we find $l=p_1(y, z)x^{n_1}+\cdots+ p_k(y,z)x^{n_k}\in l^1(G)^{\times}$ such that $$\sum_{i=1}^k2^{n_i}p_i(y,z)(1-z^{n_i}y)=0\;?$$

Here, $\forall~ 1\leq i\leq k, ~p_i(y,z)\in \mathbb{Z}G$ and $n_1<\cdots<n_k\in\mathbb{Z}$ to be determined. Note that the group element $x$ does not appear in $p_i(y, z)$.


Remarks:

1, This problem was asked previously in MSE, but no answer appeared, so I think it might be suitable for MO.

2, This problem is related to a modified Ore condition. I want to show that $l$ does not exist, suppose it exists, then I have considered the natural quotient map $\phi: G\to H=G/\langle z^2\rangle$. Note that it would induce a map $\phi: l^1(G)^{\times}\to l^1(H)^{\times}$, then $\phi(l)\in l^1(H)^{\times}$, but I still could not handle this..

$\endgroup$
4
  • $\begingroup$ why some symbols could not appear? Anyone helps fix this? $\endgroup$
    – Jiang
    May 30, 2013 at 7:38
  • 1
    $\begingroup$ @Jiang: I've edited the MathJax code. There are some issues with how it works here, but they should go away when MathOverflow moves to the SE 2.0 network sometime soon: tea.mathoverflow.net/discussion/1416/1/mathoverflow-20 $\endgroup$ May 30, 2013 at 8:00
  • $\begingroup$ How about $l = 2(1-zy) - (1-y)x$? $\endgroup$
    – user91132
    Jun 4, 2013 at 8:39
  • $\begingroup$ @Konstantin, thanks for your comment. For this $l$, we can check that $l^{-1}=sum_{n\geq 0}\frac{1}{2^{n+1}}(1-y)(1-z^ny)^{-1}(1-z^{n+1}y)^{-1}x^n\not\in l^1(G)^{\times}$ $\endgroup$
    – Jiang
    Jun 16, 2013 at 9:53

1 Answer 1

1
$\begingroup$

Omit this problem, we can take $l=z+\frac{1}{2}(-z-1)x+\frac{1}{4}x^2$, then $l^{-1}=\sum_{n\geq 0, m\geq 0}\frac{1}{2^{n+m}}x^{n+m}z^{-m-1}\in l^1(G)$.

$\endgroup$

Your Answer

By clicking “Post Your Answer”, you agree to our terms of service and acknowledge you have read our privacy policy.

Not the answer you're looking for? Browse other questions tagged or ask your own question.